Partial derivative of a two variables function, one of which dependent on the other












1












$begingroup$


I found this exercise on the book of multivariable calculus from which I'm studying:



"Find the partial derivative $frac{partial{z}}{partial{x}}$ and the total derivative $frac{text{d}z}{text{d}x}$ of $z(x,y)=e^{xy}$ where $y=phi(x)$."



Now, this to me looks like a function of a single variable $f:mathbb{R}tomathbb{R}$ and so in this case the partial derivative of $f$ with respect to $x$ and total derivative would be equivalent; in particular, I end up with something like:



$$frac{text{d}z}{text{d}x}=e^{xy}(phi(x)+xphi'(x))$$



In the solution, while the result for the total derivative is the same as mine, the partial derivative of $f$ with respect to $x$ is written as follows:



$$frac{partial{z}}{partial{x}}=ye^{xy}$$



Why is this the case?
Since the partial derivative of $f$ with respect to $x$ shows the incremental behaviour of the function as $x$ changes, shouldn't I account for the presence of $x$ in the functional representation of $y$ while computing the derivative with respect to $x$?



Sorry in advance for the super basic question :)










share|cite|improve this question









$endgroup$

















    1












    $begingroup$


    I found this exercise on the book of multivariable calculus from which I'm studying:



    "Find the partial derivative $frac{partial{z}}{partial{x}}$ and the total derivative $frac{text{d}z}{text{d}x}$ of $z(x,y)=e^{xy}$ where $y=phi(x)$."



    Now, this to me looks like a function of a single variable $f:mathbb{R}tomathbb{R}$ and so in this case the partial derivative of $f$ with respect to $x$ and total derivative would be equivalent; in particular, I end up with something like:



    $$frac{text{d}z}{text{d}x}=e^{xy}(phi(x)+xphi'(x))$$



    In the solution, while the result for the total derivative is the same as mine, the partial derivative of $f$ with respect to $x$ is written as follows:



    $$frac{partial{z}}{partial{x}}=ye^{xy}$$



    Why is this the case?
    Since the partial derivative of $f$ with respect to $x$ shows the incremental behaviour of the function as $x$ changes, shouldn't I account for the presence of $x$ in the functional representation of $y$ while computing the derivative with respect to $x$?



    Sorry in advance for the super basic question :)










    share|cite|improve this question









    $endgroup$















      1












      1








      1





      $begingroup$


      I found this exercise on the book of multivariable calculus from which I'm studying:



      "Find the partial derivative $frac{partial{z}}{partial{x}}$ and the total derivative $frac{text{d}z}{text{d}x}$ of $z(x,y)=e^{xy}$ where $y=phi(x)$."



      Now, this to me looks like a function of a single variable $f:mathbb{R}tomathbb{R}$ and so in this case the partial derivative of $f$ with respect to $x$ and total derivative would be equivalent; in particular, I end up with something like:



      $$frac{text{d}z}{text{d}x}=e^{xy}(phi(x)+xphi'(x))$$



      In the solution, while the result for the total derivative is the same as mine, the partial derivative of $f$ with respect to $x$ is written as follows:



      $$frac{partial{z}}{partial{x}}=ye^{xy}$$



      Why is this the case?
      Since the partial derivative of $f$ with respect to $x$ shows the incremental behaviour of the function as $x$ changes, shouldn't I account for the presence of $x$ in the functional representation of $y$ while computing the derivative with respect to $x$?



      Sorry in advance for the super basic question :)










      share|cite|improve this question









      $endgroup$




      I found this exercise on the book of multivariable calculus from which I'm studying:



      "Find the partial derivative $frac{partial{z}}{partial{x}}$ and the total derivative $frac{text{d}z}{text{d}x}$ of $z(x,y)=e^{xy}$ where $y=phi(x)$."



      Now, this to me looks like a function of a single variable $f:mathbb{R}tomathbb{R}$ and so in this case the partial derivative of $f$ with respect to $x$ and total derivative would be equivalent; in particular, I end up with something like:



      $$frac{text{d}z}{text{d}x}=e^{xy}(phi(x)+xphi'(x))$$



      In the solution, while the result for the total derivative is the same as mine, the partial derivative of $f$ with respect to $x$ is written as follows:



      $$frac{partial{z}}{partial{x}}=ye^{xy}$$



      Why is this the case?
      Since the partial derivative of $f$ with respect to $x$ shows the incremental behaviour of the function as $x$ changes, shouldn't I account for the presence of $x$ in the functional representation of $y$ while computing the derivative with respect to $x$?



      Sorry in advance for the super basic question :)







      multivariable-calculus derivatives partial-derivative






      share|cite|improve this question













      share|cite|improve this question











      share|cite|improve this question




      share|cite|improve this question










      asked Dec 5 '18 at 23:43









      Stefano CortinovisStefano Cortinovis

      82




      82






















          1 Answer
          1






          active

          oldest

          votes


















          0












          $begingroup$

          It is possible for $phi(x)$ to be stationary with respect to $x$ wiggles while $f(x,phi(x))$ is not stationary due to its remaining explicit dependence on $x$. When we write the partial derivative, we by-definition require that $y$ is constant, or equivalently that $phi(x)$ is constant while we wiggle x, so it doesn't contribute to the variation (hence why it gets called "partial"). The "total" variation of $f$ makes no such restriction, and thus depends on the general wiggle of both x and $phi(x)$.



          I assume you have an intuition / understanding of the following equality,
          $$
          frac{df(x,y)}{dt} = frac{partial f}{partial x} frac{partial x}{partial t} + frac{partial f}{partial y} frac{partial y}{partial t}
          $$



          Letting $x=t$ we have,
          begin{align}
          frac{df(x,y)}{dx} &= frac{partial f}{partial x} frac{partial x}{partial x} + frac{partial f}{partial y} frac{partial y}{partial x}\
          &= frac{partial f}{partial x} + frac{partial f}{partial y} frac{partial y}{partial x}
          end{align}



          Letting $y = phi(x)$ we have,
          $$
          frac{df(x,y)}{dx} = frac{partial f}{partial x} + frac{partial f}{partial y} phi'(x)
          $$



          Notice that,
          $$
          phi'(x)=0 implies frac{df}{dx}=frac{partial f}{partial x}
          $$



          The partial derivative under question is "how $f$ varies while we vary $x$ and hold $y$ constant", $frac{partial f}{partial x}$, which still makes sense even if $y=phi(x)$.



          In the case of $f(x,y) = e^{xy}$ we have,
          $$
          frac{partial f}{partial x} = ye^{xy}
          $$

          and
          begin{align}
          frac{df(x,y)}{dx} &= ye^{xy} + xe^{xy}phi'(x)\
          &= e^{xy}big{(}y + xphi'(x)big{)}
          end{align}



          which is, as you have found and we would hope, equivalent to thinking of $f$ as a function of one variable $x$ to find by "product rule",
          $$
          frac{df(x)}{dx} = e^{xphi(x)}big{(}phi(x) + xphi'(x)big{)}
          $$






          share|cite|improve this answer











          $endgroup$













          • $begingroup$
            I think I got it, thank a lot!
            $endgroup$
            – Stefano Cortinovis
            Dec 6 '18 at 12:37










          • $begingroup$
            @Stefano No problem. Generally, you can "accept" an answer to your Stack Exchange question by clicking the checkmark by the voting buttons.
            $endgroup$
            – jnez71
            Dec 6 '18 at 15:39











          Your Answer





          StackExchange.ifUsing("editor", function () {
          return StackExchange.using("mathjaxEditing", function () {
          StackExchange.MarkdownEditor.creationCallbacks.add(function (editor, postfix) {
          StackExchange.mathjaxEditing.prepareWmdForMathJax(editor, postfix, [["$", "$"], ["\\(","\\)"]]);
          });
          });
          }, "mathjax-editing");

          StackExchange.ready(function() {
          var channelOptions = {
          tags: "".split(" "),
          id: "69"
          };
          initTagRenderer("".split(" "), "".split(" "), channelOptions);

          StackExchange.using("externalEditor", function() {
          // Have to fire editor after snippets, if snippets enabled
          if (StackExchange.settings.snippets.snippetsEnabled) {
          StackExchange.using("snippets", function() {
          createEditor();
          });
          }
          else {
          createEditor();
          }
          });

          function createEditor() {
          StackExchange.prepareEditor({
          heartbeatType: 'answer',
          autoActivateHeartbeat: false,
          convertImagesToLinks: true,
          noModals: true,
          showLowRepImageUploadWarning: true,
          reputationToPostImages: 10,
          bindNavPrevention: true,
          postfix: "",
          imageUploader: {
          brandingHtml: "Powered by u003ca class="icon-imgur-white" href="https://imgur.com/"u003eu003c/au003e",
          contentPolicyHtml: "User contributions licensed under u003ca href="https://creativecommons.org/licenses/by-sa/3.0/"u003ecc by-sa 3.0 with attribution requiredu003c/au003e u003ca href="https://stackoverflow.com/legal/content-policy"u003e(content policy)u003c/au003e",
          allowUrls: true
          },
          noCode: true, onDemand: true,
          discardSelector: ".discard-answer"
          ,immediatelyShowMarkdownHelp:true
          });


          }
          });














          draft saved

          draft discarded


















          StackExchange.ready(
          function () {
          StackExchange.openid.initPostLogin('.new-post-login', 'https%3a%2f%2fmath.stackexchange.com%2fquestions%2f3027839%2fpartial-derivative-of-a-two-variables-function-one-of-which-dependent-on-the-ot%23new-answer', 'question_page');
          }
          );

          Post as a guest















          Required, but never shown

























          1 Answer
          1






          active

          oldest

          votes








          1 Answer
          1






          active

          oldest

          votes









          active

          oldest

          votes






          active

          oldest

          votes









          0












          $begingroup$

          It is possible for $phi(x)$ to be stationary with respect to $x$ wiggles while $f(x,phi(x))$ is not stationary due to its remaining explicit dependence on $x$. When we write the partial derivative, we by-definition require that $y$ is constant, or equivalently that $phi(x)$ is constant while we wiggle x, so it doesn't contribute to the variation (hence why it gets called "partial"). The "total" variation of $f$ makes no such restriction, and thus depends on the general wiggle of both x and $phi(x)$.



          I assume you have an intuition / understanding of the following equality,
          $$
          frac{df(x,y)}{dt} = frac{partial f}{partial x} frac{partial x}{partial t} + frac{partial f}{partial y} frac{partial y}{partial t}
          $$



          Letting $x=t$ we have,
          begin{align}
          frac{df(x,y)}{dx} &= frac{partial f}{partial x} frac{partial x}{partial x} + frac{partial f}{partial y} frac{partial y}{partial x}\
          &= frac{partial f}{partial x} + frac{partial f}{partial y} frac{partial y}{partial x}
          end{align}



          Letting $y = phi(x)$ we have,
          $$
          frac{df(x,y)}{dx} = frac{partial f}{partial x} + frac{partial f}{partial y} phi'(x)
          $$



          Notice that,
          $$
          phi'(x)=0 implies frac{df}{dx}=frac{partial f}{partial x}
          $$



          The partial derivative under question is "how $f$ varies while we vary $x$ and hold $y$ constant", $frac{partial f}{partial x}$, which still makes sense even if $y=phi(x)$.



          In the case of $f(x,y) = e^{xy}$ we have,
          $$
          frac{partial f}{partial x} = ye^{xy}
          $$

          and
          begin{align}
          frac{df(x,y)}{dx} &= ye^{xy} + xe^{xy}phi'(x)\
          &= e^{xy}big{(}y + xphi'(x)big{)}
          end{align}



          which is, as you have found and we would hope, equivalent to thinking of $f$ as a function of one variable $x$ to find by "product rule",
          $$
          frac{df(x)}{dx} = e^{xphi(x)}big{(}phi(x) + xphi'(x)big{)}
          $$






          share|cite|improve this answer











          $endgroup$













          • $begingroup$
            I think I got it, thank a lot!
            $endgroup$
            – Stefano Cortinovis
            Dec 6 '18 at 12:37










          • $begingroup$
            @Stefano No problem. Generally, you can "accept" an answer to your Stack Exchange question by clicking the checkmark by the voting buttons.
            $endgroup$
            – jnez71
            Dec 6 '18 at 15:39
















          0












          $begingroup$

          It is possible for $phi(x)$ to be stationary with respect to $x$ wiggles while $f(x,phi(x))$ is not stationary due to its remaining explicit dependence on $x$. When we write the partial derivative, we by-definition require that $y$ is constant, or equivalently that $phi(x)$ is constant while we wiggle x, so it doesn't contribute to the variation (hence why it gets called "partial"). The "total" variation of $f$ makes no such restriction, and thus depends on the general wiggle of both x and $phi(x)$.



          I assume you have an intuition / understanding of the following equality,
          $$
          frac{df(x,y)}{dt} = frac{partial f}{partial x} frac{partial x}{partial t} + frac{partial f}{partial y} frac{partial y}{partial t}
          $$



          Letting $x=t$ we have,
          begin{align}
          frac{df(x,y)}{dx} &= frac{partial f}{partial x} frac{partial x}{partial x} + frac{partial f}{partial y} frac{partial y}{partial x}\
          &= frac{partial f}{partial x} + frac{partial f}{partial y} frac{partial y}{partial x}
          end{align}



          Letting $y = phi(x)$ we have,
          $$
          frac{df(x,y)}{dx} = frac{partial f}{partial x} + frac{partial f}{partial y} phi'(x)
          $$



          Notice that,
          $$
          phi'(x)=0 implies frac{df}{dx}=frac{partial f}{partial x}
          $$



          The partial derivative under question is "how $f$ varies while we vary $x$ and hold $y$ constant", $frac{partial f}{partial x}$, which still makes sense even if $y=phi(x)$.



          In the case of $f(x,y) = e^{xy}$ we have,
          $$
          frac{partial f}{partial x} = ye^{xy}
          $$

          and
          begin{align}
          frac{df(x,y)}{dx} &= ye^{xy} + xe^{xy}phi'(x)\
          &= e^{xy}big{(}y + xphi'(x)big{)}
          end{align}



          which is, as you have found and we would hope, equivalent to thinking of $f$ as a function of one variable $x$ to find by "product rule",
          $$
          frac{df(x)}{dx} = e^{xphi(x)}big{(}phi(x) + xphi'(x)big{)}
          $$






          share|cite|improve this answer











          $endgroup$













          • $begingroup$
            I think I got it, thank a lot!
            $endgroup$
            – Stefano Cortinovis
            Dec 6 '18 at 12:37










          • $begingroup$
            @Stefano No problem. Generally, you can "accept" an answer to your Stack Exchange question by clicking the checkmark by the voting buttons.
            $endgroup$
            – jnez71
            Dec 6 '18 at 15:39














          0












          0








          0





          $begingroup$

          It is possible for $phi(x)$ to be stationary with respect to $x$ wiggles while $f(x,phi(x))$ is not stationary due to its remaining explicit dependence on $x$. When we write the partial derivative, we by-definition require that $y$ is constant, or equivalently that $phi(x)$ is constant while we wiggle x, so it doesn't contribute to the variation (hence why it gets called "partial"). The "total" variation of $f$ makes no such restriction, and thus depends on the general wiggle of both x and $phi(x)$.



          I assume you have an intuition / understanding of the following equality,
          $$
          frac{df(x,y)}{dt} = frac{partial f}{partial x} frac{partial x}{partial t} + frac{partial f}{partial y} frac{partial y}{partial t}
          $$



          Letting $x=t$ we have,
          begin{align}
          frac{df(x,y)}{dx} &= frac{partial f}{partial x} frac{partial x}{partial x} + frac{partial f}{partial y} frac{partial y}{partial x}\
          &= frac{partial f}{partial x} + frac{partial f}{partial y} frac{partial y}{partial x}
          end{align}



          Letting $y = phi(x)$ we have,
          $$
          frac{df(x,y)}{dx} = frac{partial f}{partial x} + frac{partial f}{partial y} phi'(x)
          $$



          Notice that,
          $$
          phi'(x)=0 implies frac{df}{dx}=frac{partial f}{partial x}
          $$



          The partial derivative under question is "how $f$ varies while we vary $x$ and hold $y$ constant", $frac{partial f}{partial x}$, which still makes sense even if $y=phi(x)$.



          In the case of $f(x,y) = e^{xy}$ we have,
          $$
          frac{partial f}{partial x} = ye^{xy}
          $$

          and
          begin{align}
          frac{df(x,y)}{dx} &= ye^{xy} + xe^{xy}phi'(x)\
          &= e^{xy}big{(}y + xphi'(x)big{)}
          end{align}



          which is, as you have found and we would hope, equivalent to thinking of $f$ as a function of one variable $x$ to find by "product rule",
          $$
          frac{df(x)}{dx} = e^{xphi(x)}big{(}phi(x) + xphi'(x)big{)}
          $$






          share|cite|improve this answer











          $endgroup$



          It is possible for $phi(x)$ to be stationary with respect to $x$ wiggles while $f(x,phi(x))$ is not stationary due to its remaining explicit dependence on $x$. When we write the partial derivative, we by-definition require that $y$ is constant, or equivalently that $phi(x)$ is constant while we wiggle x, so it doesn't contribute to the variation (hence why it gets called "partial"). The "total" variation of $f$ makes no such restriction, and thus depends on the general wiggle of both x and $phi(x)$.



          I assume you have an intuition / understanding of the following equality,
          $$
          frac{df(x,y)}{dt} = frac{partial f}{partial x} frac{partial x}{partial t} + frac{partial f}{partial y} frac{partial y}{partial t}
          $$



          Letting $x=t$ we have,
          begin{align}
          frac{df(x,y)}{dx} &= frac{partial f}{partial x} frac{partial x}{partial x} + frac{partial f}{partial y} frac{partial y}{partial x}\
          &= frac{partial f}{partial x} + frac{partial f}{partial y} frac{partial y}{partial x}
          end{align}



          Letting $y = phi(x)$ we have,
          $$
          frac{df(x,y)}{dx} = frac{partial f}{partial x} + frac{partial f}{partial y} phi'(x)
          $$



          Notice that,
          $$
          phi'(x)=0 implies frac{df}{dx}=frac{partial f}{partial x}
          $$



          The partial derivative under question is "how $f$ varies while we vary $x$ and hold $y$ constant", $frac{partial f}{partial x}$, which still makes sense even if $y=phi(x)$.



          In the case of $f(x,y) = e^{xy}$ we have,
          $$
          frac{partial f}{partial x} = ye^{xy}
          $$

          and
          begin{align}
          frac{df(x,y)}{dx} &= ye^{xy} + xe^{xy}phi'(x)\
          &= e^{xy}big{(}y + xphi'(x)big{)}
          end{align}



          which is, as you have found and we would hope, equivalent to thinking of $f$ as a function of one variable $x$ to find by "product rule",
          $$
          frac{df(x)}{dx} = e^{xphi(x)}big{(}phi(x) + xphi'(x)big{)}
          $$







          share|cite|improve this answer














          share|cite|improve this answer



          share|cite|improve this answer








          edited Dec 6 '18 at 15:58

























          answered Dec 6 '18 at 2:11









          jnez71jnez71

          2,341620




          2,341620












          • $begingroup$
            I think I got it, thank a lot!
            $endgroup$
            – Stefano Cortinovis
            Dec 6 '18 at 12:37










          • $begingroup$
            @Stefano No problem. Generally, you can "accept" an answer to your Stack Exchange question by clicking the checkmark by the voting buttons.
            $endgroup$
            – jnez71
            Dec 6 '18 at 15:39


















          • $begingroup$
            I think I got it, thank a lot!
            $endgroup$
            – Stefano Cortinovis
            Dec 6 '18 at 12:37










          • $begingroup$
            @Stefano No problem. Generally, you can "accept" an answer to your Stack Exchange question by clicking the checkmark by the voting buttons.
            $endgroup$
            – jnez71
            Dec 6 '18 at 15:39
















          $begingroup$
          I think I got it, thank a lot!
          $endgroup$
          – Stefano Cortinovis
          Dec 6 '18 at 12:37




          $begingroup$
          I think I got it, thank a lot!
          $endgroup$
          – Stefano Cortinovis
          Dec 6 '18 at 12:37












          $begingroup$
          @Stefano No problem. Generally, you can "accept" an answer to your Stack Exchange question by clicking the checkmark by the voting buttons.
          $endgroup$
          – jnez71
          Dec 6 '18 at 15:39




          $begingroup$
          @Stefano No problem. Generally, you can "accept" an answer to your Stack Exchange question by clicking the checkmark by the voting buttons.
          $endgroup$
          – jnez71
          Dec 6 '18 at 15:39


















          draft saved

          draft discarded




















































          Thanks for contributing an answer to Mathematics Stack Exchange!


          • Please be sure to answer the question. Provide details and share your research!

          But avoid



          • Asking for help, clarification, or responding to other answers.

          • Making statements based on opinion; back them up with references or personal experience.


          Use MathJax to format equations. MathJax reference.


          To learn more, see our tips on writing great answers.




          draft saved


          draft discarded














          StackExchange.ready(
          function () {
          StackExchange.openid.initPostLogin('.new-post-login', 'https%3a%2f%2fmath.stackexchange.com%2fquestions%2f3027839%2fpartial-derivative-of-a-two-variables-function-one-of-which-dependent-on-the-ot%23new-answer', 'question_page');
          }
          );

          Post as a guest















          Required, but never shown





















































          Required, but never shown














          Required, but never shown












          Required, but never shown







          Required, but never shown

































          Required, but never shown














          Required, but never shown












          Required, but never shown







          Required, but never shown







          Popular posts from this blog

          Quarter-circle Tiles

          build a pushdown automaton that recognizes the reverse language of a given pushdown automaton?

          Mont Emei